LSAT and Law School Admissions Forum

Get expert LSAT preparation and law school admissions advice from PowerScore Test Preparation.

User avatar
 Dave Killoran
PowerScore Staff
  • PowerScore Staff
  • Posts: 5852
  • Joined: Mar 25, 2011
|
#87411
Complete Question Explanation
(The complete setup for this game can be found here: lsat/viewtopic.php?f=149&t=16450)

The correct answer choice is (E)

This is the other Justify question in this game. The question stem asks you to supply a condition that forces L to have been begun in 602. From question #3, we know that placing S into 604 achieves that objective, but S in 604 is not one of the answers. However, S in 603 is one of the answers, and given that similarity, exploring the answer choice with S in 603 first is advisable.

When S is begun in 603, G and M must be begun in 601. As L cannot be in 604 or 605, L must therefore be in 602.
Accordingly, answer choice (E) is correct.

Note that in both question #3 and question #5, the correct answer addresses S, a random. While having a random appear as the correct answer in two Justify questions may at first be surprising, there is logic behind this occurrence. In games where only one random exists, sometimes placing a restriction on the one unrestricted variable creates a situation where the other variables are then forced into place. That is the situation here, and the principle is that adding a restriction to the one unrestricted variable can sometimes cause the remaining restricted variables to fall into line.
 htngo12
  • Posts: 40
  • Joined: May 19, 2016
|
#27267
Hi!

Based on the rules:
1) G-L-F
2) H no earlier than 604
3) M begun earlier than 604
4) Two monuments begun in 601, and no other began the same as the others

So M can go in 1, 2, and 3 while H can go in 4 and 5. Two slots for 1.

M/ M/ M/ H/ H/
_

1 2 3 4 5


Question # 5 states L begun in 602 (2) if which of the following is true?

My setup was:
1) G L M F H 2) G L F S H 3) G L S F H
S M M

I originally picked (B) G begun in 601 but noticed (D) M begun in 601 was an option so that rules both of them out.

Answer (A) and (B) are ruled out because the could be in either slots.

The correct answer is (E) S begun in 603. Can you explain how this is the answer?
 Adam Tyson
PowerScore Staff
  • PowerScore Staff
  • Posts: 5153
  • Joined: Apr 14, 2011
|
#27326
Your grasp on the rules here looks good - my diagram largely agrees with yours - but I cannot tell from what you shared whether you included any not-laws in your setup. The sequential rule of G-L-F, coupled with the restrictions on M and especially H, should have led to a lot of them.

For example, G certainly cannot go 4th or 5th, because it has to have at least two variables after it. But could it go 3rd? No, because then L would be 4th and F would be 5th, and then there is no room for H. G is therefore limited to going 1st or 2nd. F has fewer restrictions - it can't go 1st or 2nd, of course, but it could also go 3rd since there is room for both G and M to take the two slots at 1st position. It can also go 4th, with H 5th, or vice versa.

So now what about L? Where can it go? Not first, because it is after G, and not 5th because it is before F. 2nd seems okay (and the question here proves that it IS okay). Can L go 3rd? Sure - G, M and S would take the three slots in 1st and 2nd, in any order, and F and H would be interchangeable at 4th and 5th. What about putting L 4th? Here we have a problem - that forces F to go 5th, and we have no room for H anywhere.

We've learned now, in our initial setup and with the help of our not-laws, that L must always be 2nd or 3rd. How, then, will we come up with a solution that forces it to go 2nd in order to answer this question? Easy - put something else 3rd! That could be M, or S, or F. If anything else goes 3rd, L goes 2nd. For that matter, if anything else goes 2nd, L must go 3rd.

Armed with that as your setup, and with "M or S or F 3rd will force L to go 2nd" as your prephrase, answer E becomes a clear and easy-to-spot winner.

One last thing about this question: take note that they are not asking under what circumstances COULD L go 2nd - there are a lot of those. Rather, they are asking under what circumstances MUST L go 2nd. Answer B allows L to go 2nd, but also allows it to go 3rd, and that's why it's not the right answer. Only answer E forces L into 2nd, and that's the result we need. Careful reading of that question may be at the heart of your confusion, if it wasn't about the not-laws.
 htngo12
  • Posts: 40
  • Joined: May 19, 2016
|
#27408
Oh boy, I did't even process the not- laws with the chunks and limiting pieces with H & M. I was more focused on the placing the pieces on the diagram.

It's a technique that is necessary, but I sometimes throw that out the door under timed conditions.

Also, I understood the question as a MBT, but somehow I setup my diagram with G & M just in the first slots and circulated the remaining pieces, which led me to the could be diagram for slots 3, 4, and 5.

Based on the your reasoning with M, S, & F in slot 3 (with the possible answer of any in 3, if presented on the exam) then that forces L to go in 2.

Thank you for the suggestion of putting the answer back into the question. It is a very effective way to test if the answer is correct.
 srcline@noctrl.edu
  • Posts: 243
  • Joined: Oct 16, 2015
|
#27695
Hello Adam

I am confused on how G could not be in 603. My breakdown:

Not laws: (H) cant be 601,602,603, (M) cant be 604,605,606 (G) cant be 604,605,606, (F): cant be 601,602, (L) cant be 601.

Why cant G be placed in 603, If G was in 603 L would be in 604 F would be in 605 and H would be in 606, this still adheres to the G> L> F sequence.

Also do you mind clarifying the second half of the last rule. I think there's so many not laws that I'm getting confused.

Thankyou
Sarah
 Adam Tyson
PowerScore Staff
  • PowerScore Staff
  • Posts: 5153
  • Joined: Apr 14, 2011
|
#27697
Hey Sarah! Based on your response, I can understand why you are confused. Take another look at the scenario - how many years are there? Only 5 years - 601 through 605. You've added a 6th year to your diagram, allowing extra room for H after a GLF sequence. Oops! Careful reading of the scenario and rules will save you from those kinds of mistakes.

The last rule establishes the numerical distribution in this game - there are two variables in the 1st slot (year 601) and one variable in every other slot (602 through 605). It's a 2-1-1-1-1 distribution, with one tie at the beginning, no other ties and no repeats.

Take another look, read carefully, and then see if it doesn't all make sense now. Good luck!
 srcline@noctrl.edu
  • Posts: 243
  • Joined: Oct 16, 2015
|
#27702
Hello Adam,

Your response clears it up, I think I put 606 b/c of the 6 monuments. But your explanation of the last rule clears this game up. Timing continues to be my biggest issue on LG!

Thankyou
Sarah
User avatar
 nzLSAT
  • Posts: 21
  • Joined: Jul 03, 2021
|
#90916
How is this answer not B? G needs to go before L, so how does this not determine this answer?
 Adam Tyson
PowerScore Staff
  • PowerScore Staff
  • Posts: 5153
  • Joined: Apr 14, 2011
|
#90926
The key, nzLSAT, is that this is a question about what would FORCE L to be at 602. G at 601 would ALLOW L to be at 602, but it wouldn't force it to go there, because G before L would still allow L to be at 603. Imagine, for example, this solution:

G and S at 601
M at 602
L at 603
F at 604
H at 605

There are others, but this is enough to show that putting G at 601 doesn't mean that L must be at 602, as the question requires. G doesn't have to be immediately before L, just somewhere before L.

As I explained above, L can only ever be at 602 or else at 603. If we are looking for something that forces L to be at 602, we should be looking for something else to be at 603, leaving L with nowhere else to go but 602, and only answer E accomplishes that.
User avatar
 emilyjmyer
  • Posts: 48
  • Joined: May 11, 2022
|
#96318
Hi!

From the last reply on this post I can now see why answer B is incorrect, which is what I initially thought the answer was. But, I am confused on why my approach did not yield me the correct answer. I would like to understand why this approach is not the correct way to approach this question.

Being that the local condition says that L should be in 602 I went back to number 3 because L was in 602. In question 3 my diagram looked like
1: MG
2: L
3: F
4: S
5: H

So I skimmed the answer choices for 5 and saw that choices B and D were present in that diagram. B reads G was begun in 601. That is present in my diagram. And then, D says, M was begun in 601. I then tried to see if I could create scenarios where G or M were not in 601 that would allow L to stay in 602. I did not even think to look at answer choice E. I am not really understanding in the PowerScore explanation they knew to look at E.

Thanks!

Get the most out of your LSAT Prep Plus subscription.

Analyze and track your performance with our Testing and Analytics Package.